Toán Olympic quốc tế 1998 Tiếng Anh

51 342 0
Toán Olympic quốc tế 1998 Tiếng Anh

Đang tải... (xem toàn văn)

Tài liệu hạn chế xem trước, để xem đầy đủ mời bạn chọn Tải xuống

Thông tin tài liệu

Problem 1 A convex quadrilateral ABCD has perpendicular diagonals. The perpendicular bisectors of AB and CD meet at a unique point P inside ABCD. Prove that ABCD is cyclic if and only if triangles ABP and CDP have equal areas. Solution: Let AC and BD intersect at E. Suppose by symmetry that P is in ABE. Denote by M and N the respective feet of perpendiculars from P to AC and BD. Without assuming that PA  PB and PC  PD, we express the areas ABP and CDP as follows: 2ABP  2ABE  2PAE  2PBE  AM  PNBN  PM  AM  PNPM  BN  PMPN  AM  BN  PM  PN, 2CDP  2CDE  2PCE  2PDE  CM  PNDN  PM  CM  PNPM  DN  PMPN  CM  DN  PM  PN. Therefore 2ABP  CDP  AM  BN  CM  DN. We now assume that PA  PB and PC  PD. Suppose that ABCD is cyclic. Then the uniqueness of P implies that it must be the circumcenter. So M and N are the midpoints of AC and BD, respectively. Hence AM  CM and BN  DN,so(*) implies ABP  CDP. Conversely, suppose that ABP  CDP. Then, by (*), we have AM  BN  CM  DN.IfPA  PC, assume by symmetry that PA  PC. Then AM  CM and also BN  DN, because PB  PD. Thus AM  BN  CM  DN, a contradiction. Hence PA  PC, which implies that P is equidistant from A, B, C and D. We conclude then that ABCD is cyclic. Alternative Solution: Let AC and BD meet at E again. Assume by symmetry that P lies in BEC and denote ABE   and ACD  . The triangles ABP and CDP are isosceles. If M and N are the respective midpoints of their bases AB and CD, then PM  AB and PN  CD. Note that M, N and P are not collinear due to the uniqueness of P. Consider the median EM to the hypotenuse of the right triangle ABE.Wehave BEM  , AME  2 and EMP  90   2. Likewise, CEN  , DNE  2 and ENP  90   2. Hence MEN  90     , and a direct computation yields NPM  360   EMP  MEN  ENP  90       MEN. It turns out that, whenever AC  BD, the quadrilateral EMPN has a pair of equal opposite angles, the ones at E and P. We now prove our claim. Since AB  2EM and CD  2EN,wehaveABP   CDP  if and only if EM  PM  EN  PN,or,EM/EN  PN/PM. On account of MEN  NPM, the latter is equivalent to EMN  PNM. This holds if and only if EMN  PNM and ENM  PMN, and these in turn mean that EMPN is a parallelogram. But the opposite angles of EMPN at E and P are always equal, as noted above. So it is a parallelogram if and only if EMP  ENP;thatis,if 90   2  90   2. We thus obtain a condition equivalent to   ,ortoABCD being cyclic. Another Solution: Choose a coordinate system so that the axes lie on the perpendicular lines AC and BD, and so that the coordinates of A, B, C and D are 0,a, b,0, 0,c and d,0, respectively. By assumption, the perpendicular bisectors of AB and CD have a unique common point. Hence the linear system formed by their equations 2bx  2ay  b 2  a 2 and 2dx  2cy  d 2  c 2 has a unique solution, and it is given by x 0  cb 2  a 2   ad 2  c 2  2ad  bc , y 0  db 2  a 2   bd 2  c 2  2ad  bc . Naturally, x 0 ,y 0  are the coordinates of P. Since it is interior to ABCD, ABP and CDP have the same orientation. Then ABP  CDP if and only if ||  || This is equivalent to ax 0  by 0  ab  cx 0  dy 0  cd. Inserting the expressions for x 0 and y 0 , after the inevitable algebra work we obtain the equivalent condition ac  bda  c 2  b  d 2   0. Now, the choice of the coordinate system implies that a and c have different signs, as well as b and d. Hence the second factor is nonzero, so ABP  CDP if and only if ac  bd. The latter is equivalent to AE  CE  BE  DE, where E is the intersection point of the diagonals. However, it is a necessary and sufficient condition for ABCD to be cyclic. Problem 2 Let ABCD be a cyclic quadrilateral. Let E and F be variable points on the sides AB and CD, respectively, such that AE : EB  CF : FD.LetP be the point on the segment EF such that PE : PF  AB : CD. Prove that the ratio between the areas of triangles APD and BPC does not depend on the choice of E and F. Solution: We first assume that the lines AD and BC are not parallel and meet at S.SinceABCD is cyclic, ASB and CSD are similar. Then, since AE : EB  CF : FD, ASE and CSF are also similar, so that DSE  CSF. Moreover, we have SE SF  SA SC  AB CD  PE PF , which implies that ESP  FSP. Thus, ASP  BSP,andsoP is equidistant from the lines AD and BC. Therefore, APD : BPC  AD : BC, which is a constant. Next, assume that the lines AD and BC are parallel. Then ABCD is an isosceles trapezoid with AB  CD, and we have BE  DF.LetM and N be the midpoints of AB and CD, respectively. Then ME  NF and clearly, E and F are equidistant from the line MN. Thus P, the midpoint of EF, lies on MN. It follows that P is equidistant from AD and BC, and hence APD : BPC  AD : BC. Alternative Solution: Let AE : EB  CF : FD  a : b,wherea  b  1. Since PE : PF  AB : CD,we have d  P,AD   CD AB  CD  d  E,AD   AB AB  CD  d  F,AD  , d  P,BC   CD AB  CD  d  E,BC   AB AB  CD  d  F,BC  , where dX,YZ stands for the distance from the point X to the line YZ. Thus, we obtain APD  CD AB  CD AED  AB AB  CD AFD  a  CD AB  CD ABD  b  AB AB  CD ACD, BPC  CD AB  CD BEC  AB AB  CD BFC  b  CD AB  CD BAC  a  AB AB  CD BDC. Next, since A, B, C and D areconcyclic,wehavesinBAD  sinBCD and sinABC  sinADC. Thus, APD BPC  a  CD ABD  b  AB ACD b  CD BAC  a  AB BDC  a  CD  AB  AD  sinBAD  b  AB  CD  AD  sinADC b  CD  AB  BC  sinABC  a  AB  CD  BC  sinBCD  AD BC  a  sinBAD  b  sinADC b  sinABC  a  sinBCD  AD BC . Problem 3 Let I be the incenter of triangle ABC.LetK,L and M be the points of tangency of the incircle of ABC with AB,BC and CA, respectively. The line t passes through B and is parallel to KL. The lines MK and ML intersect t at the points R and S. Prove that RIS is acute. Solution: Since the lines KL and RS are parallel, we have, in BKR , BKR  90° A/2, KBR  90° B/2, BRK  90° C/2. Hence, by the law of sine, BR  cosA/2 cosC/2  BK. # Similarly, we have, in BLS, BLS  90° C/2, BSL  90° A/2, LBS  90° B/2. so that BS  cos  C/2  cos  A/2   BL  cos  C/2  cos  A/2   BK. # Notice now that BI  RS and IK  AB. Then, on account of (1) and (2), we obtain IR 2  IS 2  RS 2   BI 2  BR 2    BI 2  BS 2    BR  BS  2  2  BI 2  BR  BS   2  BI 2  BK 2   2IK 2  0. So,bythelawofcosine,RIS is acute. Alternative Solution: Let W be the midpoint of KL and Q the midpoint of KM. Then Q  AI, W  BI, AIKM and BIKL. We first prove that AWRI and CWSI. Since RBI  RQI  90  , the points R, B, I, Q are concyclic, which implies BRI  BQI. Also, in the right triangles AIK and BIK,wehave IQ  IA  KI 2  IW  IB, so that IQ/IW  IB/IA. Hence AIW and BIQ are similar. It follows that BRI  BQI  AWI.SinceBR  IW, this implies that RIAW.Byasimilar argument, we can prove that SICW. Thus, RIS  180  AWC. It remains to prove that AWC is obtuse. Let T be the midpoint of AC.Then2 WT  WC  WA  LC  KA.SinceLC and KA are not collinear, we have WT  LC  KA 2  CM  AM 2  AC 2 . This implies that W is inside the circle with diameter AC,andsoAWC  90°. Therefore, RIS is acute. Comment: Another proof for the fact AW  RI is as follows. Since RBI  RQI  90°, RBIQ is cyclic and its circumcircle is orthogonal to the diameter RI. Consider the inversion  with respect to the incircle of ABC.Since takes B and Q into W and A, respectively, it takes the circumcircle of RBIQ into the line AW.Since leaves the line RI invariant, we have AW  RI. Problem 4 Let M and N be points inside triangle ABC such that MAB  NAC MBA  NBC. Prove that AM  AN AB  AC  BM  BN BA  BC  CM  CN CA  CB  1. Solution: Let K be the point on the ray BN such that BCK  BMA. Note that K is outside ABC, because BMA  ACB.SinceMBA  CBK,wehaveABM  KBC;so, AB BK  BM BC  AM CK . # Then, since ABK  MBC,andAB/KB  BM/BC, we see that ABK  MBC. Hence AB BM  BK BC  AK CM . # Now we have CKN  MAB  NAC. Consequently, the points A, N, C and K are concyclic. By Ptolemy’s theorem, AC  NK  AN  CK  CN  AK,or AC  BK  BN   AN  CK  CN  AK. # From (1) and (2), we find CK  AM  BC/BM, AK  AB  CM/BM and BK  AB  BC/BM. Inserting these expressions in (3), we obtain AC  AB  BC BM  BN  AN  AM  BC BM  CN  AB  CM BM , or AM  AN AB  AC  BM  BN BA  BC  CM  CN CA  CB  1. Alternative Solution: Let the complex coordinates of A, B, C, M and N be a, b, c, m and n, respectively. Since the lines AM, BM and CM are concurrent, as well as the lines AN, BN and CN, it follows from Ceva’s theorem that sinBAM sinMAC  sinCBM sinMBA  sinACM sinMCB  1. # sinBAN sinNAC  sinCBN sinNBA  sinACN sinNCB  1. # By hypotheses, BAM  NAC and MBA  CBN. Hence BAN  MAC and NBA  CBM. Combined with (1) and (2), these equalities imply sinACM  sinACN  sinMCB  sinNCB. Thus, cos  NCM  2ACM   cosNCM  cos  NCM  2NCB   cosNCM. and hence ACM  NCB. Since BAM  NAC, MBA  CBN and ACN  MCB, the following complex ratios are all positive real numbers: m  a b  a : c  a n  a , m  b a  b : c  b n  b and m  c b  c : a  c n  c . Hence each of these equals its absolute value, and so AM  AN AB  AC  BM  BN BA  BC  CM  CN CA  CB   m  a  n  a   b  a  c  a    m  b  n  b   a  b  c  b    m  c  n  c   b  c  a  c   1. Comment: Contestants who are familiar with the notion of isogonal conjugate points may skip over the early part of the proof. Problem 5 Let ABC be a triangle, H its orthocenter, O its circumcenter, and R its circumradius. Let D be the reflection of A across BC, E be that of B across CA,andF that of C across AB. Prove that D,E and F are collinear if and only if OH  2R. Solution: Let G be the centroid of ABC,andA  , B  and C  be the midpoints of BC, CA and AB, respectively. Let A  B  C  be the triangle for which A, B and C are the midpoints of B  C  , C  A  and A  B  , respectively. Then G is the centroid and H the circumcenter of A  B  C  . Let D  , E  and F  denote the projections of O on the lines B  C  , C  A  and A  B  , respectively. Consider the homothety h with center G and ratio 1/2. It maps A, B, C, A  , B  and C  into A  , B  , C  , A , B and C, respectively. Note that A  D   BC, which implies AD : A  D   2:1 GA : GA  and DAG  D  A  G. We conclude that hD  D  . Similarly, h  E   E  and h  F   F  . Thus, D, E and F are collinear if and only if D  , E  and F  are collinear. Now D  , E  and F  are the projections of O on the sides B  C  , C  A  and A  B  , respectively. By Simson’s theorem, they are collinear if and only if O lies on the circumcircle of A  B  C  . Since the circumradius of A  B  C  is 2R, O lies on its circumcircle if and only if OH  2R. Alternative Solution: Let the complex coordinates of A, B, C, H and O be a, b, c, h and 0, respectively. Consequently, aa   bb   cc   R 2 and h  a  b  c.SinceD is symmetric to A with respect to line BC, the complex coordinates d and a satisfy d  b c  b  a  b c  b ,or  b   c   d   b  c  a    bc   b  c   0. # Since b   c    R 2  b  c  bc and bc   b  c  R 2  b 2  c 2  bc , by inserting these expressions in (1), we obtain d  bc  ca  ab a  k  2bc a . d   R 2  a  b  c  bc  R 2  h  2a  bc . where k  bc  ca  ab. Similarly, we have e  k  2ca b , e   R 2  h  2b  ca , f  k  2ab c andf   R 2  h  2c  ab . Since  dd  1 ee  1 ff  1  e  de   d  f  df   d    ba  k2ab  ab R 2  ab  h2c  abc  ca  k2ca  ca R 2  ac  h2b  abc  R 2  c  a  a  b  a 2 b 2 c 2    ck  2abc  h  2c   bk  2abc    h  2b   R 2  b  c  c  a  a  b  hk  4abc  a 2 b 2 c 2 and h   R 2 k/abc, it follows that D,EandFarecollinear 0  hk  4abc  0  hh   4R 2  OH  2R. [...]... ggn  n), and so takes distinct primes to distinct primes Hence a lower bound for g 1998  g2  3 3  37  g2g3 3 g37 is obtained when g2, g3, g37 are the three smallest primes 2, 3, 5, with g3  2 Thus, g 1998  3  2 3  5  120 for each g  S There is, however, a function g  S with g 1998  120; this proves that the minimum in question is 120 Set g1  1, and define g...  10 and 6m  10  1mod3 is already in the list Finally, we have a 4m8  9m  16, becsase no y  2mod3 is in the list The formulae for a n has been established by induction From 1998  4  499  2, we find that a 1998  9  499  3  4494 Comment: It is easy to see that x  y  3z has no solution in the set of positive integers  1 Multiply the set by 3 still has the same property Thus one takes... arbitrary n  p  1 p  2 p  k  N by setting 1 2 k gn  gp  1 p  2 p  k   gp 1   1 gp 2   2 gp k   k The conditions in (2) are satisfied 1 2 k (with a  1), so g  S Clearly, g 1998  120, which completes the proof Comment: 1 Actually, each function f considered above is of the form fn  agn, where g is a multiplicative involution of N; that is, a function satisfying (2)... is defined as follows: a 1  1 and for n  1, a n1 is the smallest integer greater than a n such that a i  a j  3a k for any i, j and k in 1, 2, 3,  , n  1, not necessarily distinct Determine a 1998 Solution: We investigate the first few values for a n , writing ‘not m’ to mean that the next value for a n cannot be m We have a 1  1, but not 2, because 1  2  3  1 We next find that a 2  3,... generating polynomials f n x does not work in itself To prove the mentioned algebraic relation presents no less difficulty than the original problem Problem 13 Determine the least possible value of f 1998 , where f is a function from the set N of positive integers into itself such that for all m, n  N, fn 2 fm  mfn 2 Solution: Denote by S the set of functions considered Let f be any of them,... consisting of positive integers  1, 3, 4, 7 Ordering this set as an increasing sequence a n The subtle point is to show that this sequence indeed is what we want, then it is easy to get the value of a 1998 from the formula for a n Problem 18 Determine all positive integers n for which there exists an integer m such that 2 n  1 is a divisor of m 2  9 Solution: First we show that if 2 n  1 divides . 1. Comment: Contestants who are familiar with the notion of isogonal conjugate points may skip over the early part of the proof. Problem 5 Let ABC be a triangle, H its orthocenter, O its circumcenter,. expressions for x 0 and y 0 , after the inevitable algebra work we obtain the equivalent condition ac  bda  c 2  b  d 2   0. Now, the choice of the coordinate system implies that a and c. being cyclic. Another Solution: Choose a coordinate system so that the axes lie on the perpendicular lines AC and BD, and so that the coordinates of A, B, C and D are 0,a, b,0, 0,c and

Ngày đăng: 05/04/2014, 00:14

Từ khóa liên quan

Tài liệu cùng người dùng

Tài liệu liên quan